Q11

 
vswamy
Thanks Received: 0
Forum Guests
 
Posts: 9
Joined: December 23rd, 2009
 
 
 

Q11

by vswamy Mon Nov 15, 2010 5:39 pm

Could you explain why question #11 has answer choice A?

I didn't choose answer choice A because you could have four french novels and one russian play and satisfy all the conditions.
User avatar
 
noah
Thanks Received: 1192
Atticus Finch
Atticus Finch
 
Posts: 1541
Joined: February 11th, 2009
 
 
 

Re: PT 32 S3 G2 French Russian

by noah Wed Nov 17, 2010 1:47 pm

My goodness how I hate this game!

There are only 3 French novels (so your hypothetical is impossible). So, if you have no Russian novels, and two of the three plays must also be out, what do you have left to fill the five slots? The basic issue is that too many things are knocked out.

French Novel
French Novel
French Novel
french/russian play
?

OUT:
Russian Novel
Russian Novel
Russian Novel
french play
french/russian play
 
rachael.swetnam
Thanks Received: 0
Forum Guests
 
Posts: 6
Joined: November 30th, 2010
 
 
 

Re: PT 32 S3 G2 French Russian

by rachael.swetnam Tue Nov 30, 2010 10:46 pm

wow, it's so cool of you to post this for free. thanks :)
User avatar
 
noah
Thanks Received: 1192
Atticus Finch
Atticus Finch
 
Posts: 1541
Joined: February 11th, 2009
 
 
 

Re: PT 32 S3 G2 French Russian

by noah Thu Dec 02, 2010 10:08 am

rachael.swetnam Wrote:wow, it's so cool of you to post this for free. thanks :)

Thanks! We think it's a good way to let people know that we're serious geeks.

We hope it's helpful.
 
fch4mat
Thanks Received: 0
Forum Guests
 
Posts: 4
Joined: January 16th, 2011
 
 
 

Re: Q11

by fch4mat Sun Jan 16, 2011 9:17 pm

When I read that question, I understood it to be no Russian novels and one Russian play were selected. Either way, it would not have been enought French works.

Thanks so much for the site! There are those of us out here who appreciate the serious geeks. I guess you could say that we are trying to be serious LSAT geeks as well. Thanks again for the site! :D
 
amy
Thanks Received: 0
Forum Guests
 
Posts: 1
Joined: October 24th, 2010
 
 
 

Re: Q11

by amy Mon Apr 16, 2012 12:20 pm

Wht is "A" correct? In the stimulus it states that AT LEAST THREE novels must be selected. However, in questions 11 answers B - E only have one or two novels selected. How are these answers correct?
 
timmydoeslsat
Thanks Received: 887
Atticus Finch
Atticus Finch
 
Posts: 1136
Joined: June 20th, 2011
 
 
trophy
Most Thanked
trophy
First Responder
 

Re: Q11

by timmydoeslsat Mon Apr 16, 2012 1:21 pm

The question stem asks us what could be true about the works.

In choice B, for example, it could be true that exactly one R(N) and F(P) F(P) are selected.

It would require us to have both F(N) F(N) included in the work so as to comply with the rule of there being at least as many French novels as Russian novels.

These answer choices are simply giving us facts about the selections, not the entire lineup.

A cannot be true. It is not possible for there to be 0 R(N) and exactly 1 play.

There has to be at least 3 novels, so we would have to use all three French novels. Then we can only select one play. That is only 4 works selected. We must have at least 5. This is something that must be false.
User avatar
 
ManhattanPrepLSAT1
Thanks Received: 1909
Atticus Finch
Atticus Finch
 
Posts: 2851
Joined: October 07th, 2009
 
 
 

Re: Q11

by ManhattanPrepLSAT1 Wed Oct 17, 2012 12:38 pm

Image
 
jeanh93
Thanks Received: 0
Vinny Gambini
Vinny Gambini
 
Posts: 1
Joined: July 28th, 2015
 
 
 

Re: Q11

by jeanh93 Tue Jul 28, 2015 8:16 pm

I didn't choose (A) due to the contrapositive of Fp & Fp --> ~Rp, which is Rp --> ~Fp or Fp (if Rp is selected, then either one of the Fp cannot be selected).
This enabled me to come up with the following: (A) Rp --> Fn + Fn + Fn + Fp, thus fulfilling the conditional statements. Can anyone tell me why this is wrong?